Autor Tema: ¿Qué es lo correcto?

0 Usuarios y 1 Visitante están viendo este tema.

03 Octubre, 2019, 12:31 pm
Respuesta #480

minette

  • $$\Large \color{#5b61b3}\pi\,\pi\,\pi\,\pi\,\pi$$
  • Mensajes: 1,037
  • Karma: +0/-5
  • Sexo: Femenino
Hola

Concrétame por favor qué es lo que te tengo que aclarar.

Saludos.

03 Octubre, 2019, 12:48 pm
Respuesta #481

Luis Fuentes

  • el_manco
  • Administrador
  • Mensajes: 55,996
  • País: es
  • Karma: +0/-0
Hola

Concrétame por favor qué es lo que te tengo que aclarar.

Nada. Simplemente que eso último que has dicho sobre el "honor a Wiles" no aporta nada nuevo. Mi respuesta a lo anterior sigue siendo la misma:

Hola

¿Puede alguien demostrar que en una terna viable \( 3a^n>b^n \)?

Pues insisto en que no estoy cien por cien seguro a que llamas terna viable.

Terna viable.- Es aquella que podría dar lugar a \( a^n+b^n = c^n \). Una terna viable cumple \( a+b>c \) , cuando \( c>b>a \).

Dices que "podría". Pero eso es una vaguedad. En realidad sabemos (por que lo probó Wiles) que para \( n\geq 3 \) ninguna terna de naturales cumple esa ecuación; por tanto no hay ternas viables porque ninguna tripleta de naturales dará lugar a esa ecuación. En ese sentido cualquier afirmación que hicieses sobre una tal tripleta sería cierta, porque sería una afirmación sobre los elementos del conjunto vacío.

Por lo demás yo no veo ninguna prueba sencilla ni directa de que una tal terna tenga que cumplir \( 3a^n>b^n \). Desde luego para números reales, hay ternas tales que \( a^n+b^n=c^n \) y sin embargo \( 3a^n\leq b^n. \)

Si tu no tienes nada más que añadir, yo tampoco.

Saludos.

18 Octubre, 2019, 06:08 pm
Respuesta #482

minette

  • $$\Large \color{#5b61b3}\pi\,\pi\,\pi\,\pi\,\pi$$
  • Mensajes: 1,037
  • Karma: +0/-5
  • Sexo: Femenino
Hola

Dadas estas dos diferencias:

\( c^n-2a^n \)  ;  \( b^n -a^n \)

con \( c>b>a \) naturales y \( n= \) natural > 2

¿necesita demostración que estas diferencias aumentan al aumentar \( n \) ?

Saludos.

18 Octubre, 2019, 06:20 pm
Respuesta #483

Luis Fuentes

  • el_manco
  • Administrador
  • Mensajes: 55,996
  • País: es
  • Karma: +0/-0
Hola

Dadas estas dos diferencias:

\( c^n-2a^n \)  ;  \( b^n -a^n \)

con \( c>b>a \) naturales y \( n= \) natural > 2

¿necesita demostración que estas diferencias aumentan al aumentar \( n \) ?

Pues... si. Es decir cualquier afirmación en matemáticas necesita demostración; ahora en este caso es muy fácil la prueba, y si usases en algún momento ese hecho en algún desarrollo más largo en un contexto de investigación académica nadie te exigiría que lo probases porque se daría por obvio.

Spoiler
Por ejemplo basta tener en cuenta en general que:

\( x^n-y^n=(x-y)(\color{blue}x^{n-1}+x^{n-2}y+x^{n-3}y^2+\ldots+y^{n-1}\color{black}) \)

Y el término en azul es claramente creciente en \( n \) porque todos los términos son positivos.
[cerrar]

Saludos.

23 Octubre, 2019, 05:41 pm
Respuesta #484

minette

  • $$\Large \color{#5b61b3}\pi\,\pi\,\pi\,\pi\,\pi$$
  • Mensajes: 1,037
  • Karma: +0/-5
  • Sexo: Femenino
Hola

Dada esta desigualdad siendo \(  b>a \):
 

\( 3a^{^{2n}}+2b^{n}a^{n}\neq b^{2n} \)
 

\( 3a^{2n}\neq b^{2n}-2b^{n}a^{n} \)
 

\( 3a^{n}.a^{n}\neq b^{n}(b^{n}-2a^{n}) \)
 

factor  \( a^{n}<  \) factor \( b^{n} \)
 

factor  \( 3a^{n}?  \) factor  \( (b^{n}-2a^{n}) \)
 

\( 3a^{n}+2a^{n}?b^{n}\rightarrow5a^{n}>b^{n} \)
 

¿Cabe colegir que \( 3a^{2n}+2b^{n}a^{n}>b^{2n} \) ?

Saludos.

24 Octubre, 2019, 09:45 am
Respuesta #485

Luis Fuentes

  • el_manco
  • Administrador
  • Mensajes: 55,996
  • País: es
  • Karma: +0/-0
Hola

Dada esta desigualdad siendo \(  b>a \):
 

\( 3a^{^{2n}}+2b^{n}a^{n}\neq b^{2n} \)
 

\( 3a^{2n}\neq b^{2n}-2b^{n}a^{n} \)
 

\( 3a^{n}.a^{n}\neq b^{n}(b^{n}-2a^{n}) \)
 

factor  \( a^{n}<  \) factor \( b^{n} \)
 

factor  \( 3a^{n}?  \) factor  \( (b^{n}-2a^{n}) \)
 

\( 3a^{n}+2a^{n}?b^{n}\rightarrow5a^{n}>b^{n} \)
 

¿Cabe colegir que \( 3a^{2n}+2b^{n}a^{n}>b^{2n} \) ?

Si lo que preguntas es si del hecho de que \( b>a \) puede deducirse que \( 3a^{2n}+2b^na^n>b^{2n} \), la respuesta es NO, porque de hecho es falso.

Por ejemplo toma \( n=3 \), \( a=10 \), \( b=17 \).

Tampoco puede asegurarse la desigualdad opuesta. Por ejemplo toma  \( n=3 \), \( a=10 \), \( b=13 \).

Saludos.

30 Octubre, 2019, 12:08 pm
Respuesta #486

minette

  • $$\Large \color{#5b61b3}\pi\,\pi\,\pi\,\pi\,\pi$$
  • Mensajes: 1,037
  • Karma: +0/-5
  • Sexo: Femenino
Hola

Siendo \( c>a \) trato de demostrar que en \( c^{2n}? 4c^na^n  \), el interrogante sólo puede ser \( = \) si nos ceñimos a enteros positivos.

Si \( c^{2n}=4c^na^n \) ; \( c^{2n}-4c^na^n=0 \) ; \( c^n(c^n-4a^n)=0 \) con lo cual \( c_1^n=0 \) ; \( c_2^n=4a^n \).

Si \( c^{2n}>4c^na^n \) ; \( c^{2n}+t=4c^na^n \)

Despejando \( c^n \)

\( c^n=2a^n\pm{}\sqrt[ ]{4a^{2n}-t} \)

con lo cual la raiz sólo puede ser entera si \( t=0 \) y llegamos a \( c_1^n=0 \) y \( c_2^n=4a^n \)

Lo mismo para el caso \( c^{2n}<4c^na^n \)

Saludos.

30 Octubre, 2019, 01:25 pm
Respuesta #487

Luis Fuentes

  • el_manco
  • Administrador
  • Mensajes: 55,996
  • País: es
  • Karma: +0/-0
Hola

Siendo \( c>a \) trato de demostrar que en \( c^{2n}? 4c^na^n  \), el interrogante sólo puede ser \( = \) si nos ceñimos a enteros positivos.

No puedes pretender probar tal cosa porque es FALSA. Por ejemplo basta tomar \( a=1 \), \( c=10 \) y \( n=3. \)

De hecho,... ¡obviamente!.... sólo se tiene la igualdad si \( c^n=4a^n \). Es decir fijado \( n \) y dado un valor de \( a \), existe un único valor de \( c \) para el cuál se tiene la igualdad.

Saludos.

30 Octubre, 2019, 06:55 pm
Respuesta #488

minette

  • $$\Large \color{#5b61b3}\pi\,\pi\,\pi\,\pi\,\pi$$
  • Mensajes: 1,037
  • Karma: +0/-5
  • Sexo: Femenino
Hola

Perdona Luis por ser tan corta de entendimiento.

Pero tengo la impresión que los dos párrafos de tu respuesta 487 se contradicen.

O que tu respuesta 487 avala la mía 486.

Saludos.

31 Octubre, 2019, 07:37 am
Respuesta #489

Luis Fuentes

  • el_manco
  • Administrador
  • Mensajes: 55,996
  • País: es
  • Karma: +0/-0
Hola

Pero tengo la impresión que los dos párrafos de tu respuesta 487 se contradicen.

No hay ninguna contradicción.

1) Primero digo que es FALSO que si \( c>a \) sólo se cumpla la igualdad en \( c^{2n}=4c^na^n \), porque de hecho pongo un ejemplo donde no se cumple.

2) Después preciso más y digo que de hecho esa igualdad sólo se cumple cuando \( c^n=4a^n. \)

Citar
O que tu respuesta 487 avala la mía 486.

Pues en principio no la avala, porque como he dicho al principio de tu mensaje 486 afirmas algo que es falso. Se me ocurre que te haya entendido mal y que lo único que quisieras decir es que \( c^{2n}=4c^na^n \), sólo cuando \( c^n=4a^n \). Eso es tan obvio, como cierto, como inútil.

Saludos.